Guys please help!! Show your work! Any answers help

Guys Please Help!! Show Your Work! Any Answers Help

Answers

Answer 1

Answer:

I only know how to do the first and second one i'm not sure about the rest:  V=Lwh

Step-by-step explanation:

so, (6)(6)(8)= 288ft^3

and (12)(16)(18)= 3456m^3

Hope i have helped you in some way!


Related Questions

Use long division to find the quotient below.

(15x^3+2x^2-75)÷(3x-5)​

Answers

Answer:

  5x^2 + 9x + 15.

Step-by-step explanation:

There is no term in x so we add  one (0x).

            5x^2 + 9x + 15  <------------Quotient.

          --------------------------------------

3x - 5 )  15x^3 + 2x^2 + 0 x  - 75

            15x^3 - 25x^2

           --------------------

                         27x^2 + 0x

                         27x^2 - 45x

                         ------------------

                                      45x - 75

                                       45x - 75

                                         ..........

use natural logarithms to solve the equation 5e^3x+7=21​

Answers

Answer:

5e^(3x+7)=21

e^(3x+7)=4.2

(3x+7)lne=ln4.2

lne=1

3x+7=1.435

3x= -5.565

x= -1.855

To solve the equation [tex]5e^{3x}+7=21[/tex], isolate the exponential term, take the natural logarithm of both sides, and then solve for x. This results in x ≈ 0.3362.

To solve the equation [tex]5e^{3x}+7=21[/tex], follow these steps:

First, isolate the exponential term by subtracting 7 from both sides:

[tex]5e^{3x} = 14[/tex]

Next, divide both sides of the equation by 5:

[tex]e^{3x} =\frac{14}{5}[/tex]

Now, take the natural logarithm (ln) of both sides:

[tex]ln(e^{3x}) = ln(\frac{14}{5})[/tex]

Since the natural logarithm and the exponential function are inverse operations, the left side simplifies to:

[tex]{3x} = ln(\frac{14}{5})[/tex]

Finally, solve for x by dividing by 3:

[tex]x =\frac{1}{3}\times ln(\frac{14}{5})[/tex]

Using a calculator, this results in:

x ≈ 0.3362

fourteen of the 100

digital video recorders​ (DVRs) in an inventory are known to be defective. What is the probability that a randomly selected item is​ defective?

Answers

Answer:

0.14

Step-by-step explanation:

Fourteen of the 100  digital video recorders​ (DVRs) in an inventory are known to be defective. It means that the probability that a randomly selected item is defective is: [tex]\frac{14}{100} = 0.14[/tex].

Select the correct answer from the drop-down menu.
Divide the binomial by the monomial to find the quotient.
48r4y – 725672
-12.12y

Answers

Answer:

[tex]-(4x^2)+6(x^4)y[/tex]

Step-by-step explanation:

Here apply the law of indices where ;

[tex]A^x*A^y=A^{x+y}[/tex]

Given;

You should first simplify the numerator using the denominator as a factorThen cancel the like termFollowed by further factorization

[tex]\frac{48x^4y-72x^6y^2}{-12x^2y} \\\\\\\frac{-12x^2y(4x^2-6x^4y^2)}{-12x^2y} \\\\\\=-4x^2+6x^4y\\\\\\=-(4x^2)+6(x^4)y[/tex]

The scores on a test are normally distributed with a mean of 80 and a standard deviation of 16. What is the score that is 2 standard deviationsdeviations aboveabove the​ mean?

Answers

The deviation from 80 of about 16 is normal.

So the lowest is usually 80 - 16 = 64 and the highest is 80 + 16 = 96

But now we have a "deviation" that is 2 standard deviations above the mean score.

So 80 + 16 + 16 = 112 is this score.

Hope this helps.

r3t40

Final answer:

To calculate the score that is 2 standard deviations above the mean of 80 with a standard deviation of 16, you add 2 times the standard deviation to the mean, resulting in a score of 112.

Explanation:

To find the score that is 2 standard deviations above the mean on a test with a mean of 80 and a standard deviation of 16, we use the formula:

Score = Mean + (Number of Standard Deviations × Standard Deviation)

So, plugging in the numbers we get:

Score = 80 + (2 × 16)

Score = 80 + 32

Score = 112

The desired score is therefore 112, which is 2 standard deviations above the mean of 80.

Which of the following is a factor of 2x^4 + 20x^3 + 50x^2?


A. 2x^3
B. x^4
C. x+4
D. x+5

Answers

Answer:

D. x + 5

Step-by-step explanation:

2x^4 + 20x^3 + 50x^2

= 2x^2 (x^2 + 10x + 25)

= 2x^2 (x + 5)^2

= 2x^2 (x + 5) (x + 5)

Answer

D. x + 5

For this case we have the following expression:

[tex]2x ^ 4 + 20x ^ 3 + 50x ^ 2[/tex]

It is observed that we can extract common factor [tex]2x ^ 2[/tex], since it is common in the three terms:

[tex]2x ^ 2 (x ^ 2 + 10x + 25) =[/tex]

If we factor the expression into parentheses, we must look for two numbers that add 10 and multiply 25. These are: 5 and 5.

Rewriting the expression we have:

[tex]2x ^ 2 ((x + 5) (x + 5))[/tex]

Thus, one of the factors of the original expression is[tex]x + 5[/tex].

Answer:

Option D

what is the area of the triangle? (sorry if its sideways)​

Answers

Answer:

6

Step-by-step explanation:

2 x 6 = 12/2 = 6

Answer:

A = 6 units ^2

Step-by-step explanation:

The area of a triangle is found by

A = 1/2 bh  where b is the length of the base and h is the height

A = 1/2 (6) *2

A = 6 units ^2

What is the chance that a person randomly selected off the street was born in May
8%
12%
20%
30%

Answers

Answer: Eight percent.

Step-by-step explanation: In order to find this, we have to use the probability formula, which is:

The Event We Are Looking For/All of the Events.

In this case, plugging the numbers into the formula we get

May(1)/All the months in the year, including May (12).

Diving this, our answer will be 0.08333..., or rounded up to 0.8

However, since we are only looking for one specific event, we can take the number 100, and divide it by all the events. 12/100 is 8.333..., or rounded to 8.

Final answer:

The chance that a person randomly selected off the street was born in May is about 8%, assuming birth rates are evenly distributed throughout the year.

Explanation:

The question, "What is the chance that a person randomly selected off the street was born in May?" is one that pertains to the subject of probability in Mathematics. Since there are typically 12 equally likely months that a person could be born in, and assuming there is no significant variation in birth rates by month, the chance that someone chosen at random off the street was born in May is 1 out of 12 months.

Thus, to calculate the probability, you would divide 1 by 12, which equals approximately 0.0833. To express this as a percentage, we multiply by 100, yielding an 8.33% chance. Therefore, the closest answer among the provided options is 8%.

What’s the value of x

Answers

Answer:

x = 25

Step-by-step explanation:

The 2 given angles form a straight angle and are supplementary, that is

2x + 2 + 5x + 3 = 180

7x + 5 = 180 ( subtract 5 from both sides )

7x = 175 ( divide both sides by 7 )

x = 25

Complete the solution of the equation. Find the value of y when x equals -7.
-4x-9y=-26

Answers

Answer:

y = 6

Step-by-step explanation:

Put x = -7 to the equation -4x - 9y = -26, and solve it for y:

-4(-7) - 9y = -26

28 - 9y = -26            subtract 28 from both sides

-9y = -54           divide both sides by (-9)

y = 6

5. (03.02)
If g(x) = x2 + 3, find g(4). (2 points)
16
19
8
11​

Answers

Answer:

19

Step-by-step explanation:

g(x)=x^2+3

g(x)=(4^2)+3

g(x)=16+3=19

if g of x = x squared + 3, then 4 squared = 16, plus 3 = 19

Answer:

option(B)  g(4) =19.

Step-by-step explanation:

Given:If g(x) = [tex]x^{2}[/tex] + 3.

To find: g(4).

Solution: We have given that  

g(x) = [tex]x^{2}[/tex] + 3.

for x=4.

g(4) = [tex]4^{2}[/tex] + 3.

g(4) = 16+3

g(4) =19.

Therefore, option(B)  g(4) =19.

Pam is playing with red and black marbles. The number of red marbles she has is three more that twice the number of black marbles she has. She has 42 marbles in how. How many red marbles does Pam have?

Answers

Answer:

29 red marbles

Step-by-step explanation:

Call R the number of red marbles and B the number of black marbles.

According to the second sentence, R = 2B + 3.

According to the third sentence, R + B = 42.

Now we have 2 equations, 2 unknowns. Sub the first into the second since it already has R isolated:

2B + 3 + B = 42

3B = 39

B = 13

Now sub this into either of the original equations (I'll use the first):

R = 2(13) + 3

R = 29

Sketch the graph of y = (x - 3)2 - 25, then select the graph that corresponds
to your sketch.

Answers

Answer:

Option C. Graph C

Step-by-step explanation:

we have

[tex]y=(x-3)^{2}-25[/tex]

This is the equation of a vertical parabola open upward (vertex is a minimum)

The vertex is the point (3,-25)

therefore

The graph is C

The graph in the attached figure

Answer:c

Step-by-step explanation:

Workers have packed 1,400 glasses in 7 boxes. If they pack 3 more boxes, how many glasses will they have packed in all?

Answers

1box = 1400/7 = 200

200×3=600

1400+600=2000

Answer:

2000

Step-by-step explanation:

Given :Workers have packed 1,400 glasses in 7 boxes.

To Find :If they pack 3 more boxes, how many glasses will they have packed in all?

Solution:

Workers packed no. of glasses in 7 boxes = 1400

Workers packed no. of glasses in 1 box = [tex]\frac{1400}{7}[/tex]

Workers packed no. of glasses in 3 boxes = [tex]\frac{1400}{7} \times 3[/tex]

                                                                      = [tex]600[/tex]

So, initially they packed 1400 glasses

If they pack 3 more boxes so, the pack 600 glasses more

So, The total no. of glasses have packed by workers = 1400+600 = 2000

Hence they have packed 2000 glasses in all.

A family is planning a wedding celebration in a gazebo. They want to know the area so that they can determine how many people can comfortably fit inside. They take some measurements and find that each side is 8 ft long and the apothem is 9.7 ft. What is the area of the enclosed gazebo?​

Answers

Answer:

The area of the enclosed gazebo is [tex]310.4\ ft^{2}[/tex]

Step-by-step explanation:

I assume that is a regular octagon (eight equal aides)

we know that

The area of a regular polygon is equal to

[tex]A=\frac{1}{2}(P)(a)[/tex]

where

P is the perimeter of the polygon

a is the apothem

Find the perimeter P (the octagon has 8 sides)

[tex]P=8(8)=64\ ft[/tex]

[tex]a=9.7\ ft[/tex]

substitute

[tex]A=\frac{1}{2}(64)(9.7)[/tex]

[tex]A=310.4\ ft^{2}[/tex]

Use numerals instead of words. If necessary use/ for the fraction bar

Answers

Answer:

10

Step-by-step explanation:

Range=big-small=34-16=18

Interquartile range=big number in box-small number in box= 29-21=8

The different between the two is 18-8=10

Answer:

The difference of range and interquartile range is 10.

Step-by-step explanation:

Consider the provided information.

From the box plot we can identify:

The lowest value is 16

The first quartile is 21

The 2nd quartile or median is 26

The 3rd quartile is 29

The highest value is 34

Range is the difference of the highest value and lowest value of the data set.

Range = 34 - 16 = 18

Interquartile range is the difference of the 3rd quartile from the 1st quartile.

IQR: 29 - 21 = 8

Thus, the difference of range and interquartile range is:

18 - 8 = 10

Hence, the difference of range and interquartile range is 10.

What is the point-slope form of a line with slope 2 that contains the point (1,3)?

A. y-1 = 2(x-3)
B. y+ 3 = -2(x - 2)
c. y + 3 = 2(x+1)
D. Y-3 = 2(x - 1)​

Answers

Y-3= 2(x-1)

The answer is D


Hope this helps!

A farmer wants to put a fence around a garden in the shape of a square. The fence posts are placed every 2 meters. One side of the garden is 10m long. How many posts does the farmer need?

Answers

Answer:

The farmer would need 80 posts.

Step-by-step explanation:

If one side of the garden is 10m long, and the garden is a square, we can assume that all 4 sides will be 10m. That makes it 40m in total. Times 40×2 and you get 80.

Three sisters went shopping for Mother’s Day. Each sister bought a gift for their mom. Maggie spent 3 times as much as Karen. Karen spent half as much as Jasmine. All together, they spent $60. Then, solve your equation to determine how much each sister spent on their gift.

Answers

Answer:

Let j = amount Jasmine spent, k = amount Karen spent, and m = amount Maggie spent.

m = 3k, k = (1/2)j

j + k + m = $60

2k + k + 3k = $60

6k = $60

k = $10, m = $30, j = $20

Jasmine spent $20, Karen spent $10, and Maggie spent $30.

Two companies allow you to pay monthly for your food truck permits. Company A charges a one time fee of $150 and $45 per month. Company B charges a one time fee of $125 and $50 per month.

Answers

The simultaneous Equations for both total costs are;

Company A: y = 45x + 150

Company B: y = 50x + 125

Thus, both companies charge the same amount of money for food truck permits for 5 months

How to find the equation of the total charges?

We are told that;

Company A charges a one time fee of $150 and $45 per month.

Company B charges a one time fee of $125 and $50 per month.

Thus, using the concept of the equatiom of a line In slope intercept form, we have:

Company A: y = 45x + 150

Company B: y = 50x + 125

let's use the number 5 for x as an example

45(5) + 150 = 375

50(5) + 125 = 375

So both companies charge the same amount of money for food truck permits for 5 months.

Complete question is;

Two companies allow you to pay monthly for your food truck permits. Company A charges a one time fee of $150 and $45 per month. Company B charges a one time fee of $125 and $50 per month. Write an equation or a system of equations and explain what each solution tells you about the situation

evaluate x -2 y 0 / x 3 y -2 when x = 2 and y = 5

Answers

Answer:

5/8

Step-by-step explanation:

first plug in for x & y, then solve. you should get 5/8

2 -2 +5 +0 / 2 +3 +5 -2

5/8

Answer:

125/32

Step-by-step explanation:

Please help!!!!! Thanks

Answers

Answer:

Look to the attached file

Step-by-step explanation:

Find the additive inverse of 6+4i

Answers

[tex]0-(6+4i)=0-6-4i=-6-4i[/tex]

The additive inverse of the complex number 6+4i is -6-4i because it negates both the real and imaginary parts, resulting in a sum of zero when added to the original number.

The additive inverse of a complex number is a number that, when added to the original number, yields a sum of zero. For the complex number 6+4i, its additive inverse is found by changing the sign of both the real and the imaginary parts. Therefore, the additive inverse of 6+4i is -6-4i.

Use the graph of f(x) to evaluate the following:

Answers

Answer:

[tex]\large\boxed{\dfrac{1}{2}}[/tex]

Step-by-step explanation:

[tex]\text{The average rate of change of function}\ f(x)\\\text{over the interval}\ a\leq x\leq b\ \text{is given by this expression:}\\\dfrac{f(b)-f(a)}{b-a}.\\\\\text{Read from graph the values of function for}\ x=3\ \text{and}\ x=5.\\(look\ at\ the\ picture)\\\\f(3)=1,\ f(5)=2\\\\\text{Substitute:}\\\\\dfrac{f(5)-f(3)}{5-3}=\dfrac{2-1}{2}=\dfrac{1}{2}[/tex]

Rationalize the denominator and simplify. 6 /5-3

Answers

Final answer:

To rationalize the denominator and simplify the expression 6 / (5-3), multiply the numerator and denominator by 1/2. Then, simplify the expression to get 3/1 = 3.

Explanation:

To rationalize the denominator and simplify the expression 6 / (5-3), we can start by multiplying the numerator and denominator by a skillfully chosen factor. In this case, we can choose 1/2 as the factor.

The numerator becomes 6 * 1/2 = 3, and the denominator becomes (5-3) * 1/2 = 2/2 = 1.

Therefore, the simplified expression is 3/1 = 3.

What is the solution to the inequality X squared <49

Answers

[tex]x^2<49\\x<7 \wedge x>-7\\x\in(-7,7)[/tex]

Please help me this is urgent 15 points and brainiest
If m∠EQD=(5x+2)°, what is the value of x?

Answers

Answer:  x = 10

=====================================================

Explanation:

The angle EQD is an inscribed angle that cuts off the arc from E to D (the shortest path). Note how central angle ECD also cuts off this same arc. By the inscribed angle theorem, we know that

inscribed angle = (1/2)*(central angle)

angle EQD = (1/2)*(angle ECD)

We can multiply both sides by 2 and flip the equation to get

angle ECD = 2*(angle EQD)

Now replace "angle EQD" with 5x+2

angle ECD = 2*(5x+2)

2*(5x+2) = angle ECD

Next, replace "angle ECD" with 104 as this is the measure of this central angle.

2*(5x+2) = angle ECD

2*(5x+2) = 104

From here, we solve for x

2*(5x+2) = 104

2*5x + 2*2 = 104

10x + 4 = 104

10x+4-4 = 104-4 ..... subtracting 4 from both sides

10x = 100

10x/10 = 100/10 ...... dividing both sides by 10

x = 10

solve for x in the equation
I need help ASAP​

Answers

The answer is D

You just move all numbers to the left and set it equal to 0
And then you add all same variables together
After that you use the quadratic formula

Answer:

D

Step-by-step explanation:

Given

2x² + 3x - 7 = x² + 5x + 39

Subtract x² + 5x + 39 from both sides

x² - 2x - 46 = 0 ← in standard form

with a = 1, b = - 2, c = - 46

Solve for x using the quadratic formula

x = (- (- 2) ±[tex]\sqrt{(-2)^2-(4(1)(-46)}[/tex] ) / 2

  = ( 2 ± [tex]\sqrt{4+184}[/tex] ) / 2

  = ( 2 ± [tex]\sqrt{188}[/tex] ) / 2

  = ( 2 ± 2[tex]\sqrt{47}[/tex] ) / 2

  = 1 ± [tex]\sqrt{47}[/tex]

What is the point-slope form of a line that has a slope of 1/2 and passes through point (–7, 2)?

Answers

Answer:

D

Step-by-step explanation:

The equation of a line in point- slope form is

y - b = m(x - a)

where m is the slope and (a, b) a point on the line

here m = [tex]\frac{1}{2}[/tex] and (a, b) = (- 7, 2), hence

y - 2 = [tex]\frac{1}{2}[/tex] (x - (- 7)) → D

y - 2 = [tex]\frac{1}{2}[/tex](x + 7)

The point-slope form of the equation will be [tex]y-2 = \dfrac{1}{2}(x-(-7))[/tex]. The correct option is D.

In mathematics, slope refers to the measure of the steepness or inclination of a line or a curve. It quantifies the rate at which one variable changes with respect to another variable. The slope is commonly denoted by the letter "m."

Given that the slope of the line is 1/2. The point through which the line is passing is (-7,2).

The general form of the equation of the line passing through the point (-7,2) and the slope is 1/2 can be written as,

[tex]y-y_1 = m(x-x_1)[/tex]

The equation of the line in point-slope form can be written as,

[tex]y - y_1 = m(x-x_1)\\y-2=\dfrac{1}{2}(x-(-7))[/tex]

To know more about slope follow

https://brainly.com/question/29421841

#SPJ2

Which unit of measure would be appropriate for the volume of a cylinder that is 8 meters tall and has a radius of 4 meters

Answers

Answer: cubic meters : m³

Step-by-step explanation:

Cylinder volume is the product of area of the base by height.

Area of the base is the product of π·radius² = square meters : m²

Volume = square meters·meters =  m²·m = cubic meters : m³

[tex]\textit{\textbf{Spymore}}[/tex]

Other Questions
2 PolnisWhich of the following is an advantage of an in vitro experiment?Please help me What is the ratio for the surface areas of the cones shown below, given thatthey are similar and that the ratio of their radil and altitudes is 4:3?23 A rectangle has a width of 9 units and a length of 40 units What is the length of the diagonal? A value that describes how heavy an object is and is related to the force of gravity is behavioral adaptation can be either instinctual or Whats half of 1/3????? lydia graphed triangle LMN at the coordinates L (0, 0), M(2, 2) and N(2, -1). She thinks triangle LMN is a right triangle. Is lydias assertion correct? Which of these metals are magnetic? atiana, conoces a las hermanas de Andrs?Tatiana: S, yo _____ conozco.A. los B. la C. lo D. las Identify mABC. HELP ASAP!! A radio tower is located 300 feet from a building. From a window in the building, a person determines that the angle of elevation to the top of the tower is 42 and that the angle of depression to the bottom of the tower is 37. How tall is the tower? Perform the indicated operation. 9z^3/16xy . 4x/27z^3 Does sand/silt have any internal structures? The solution set for 6a2 - a -5 = 0 is (I think the answer is A, am I right?)Most regulatory signs are _____.A. octagonal and red or whiteB. circular and blue or greenC. rectangular and red or whiteD. rectangular and yellow What is the factored form of the equation 2(x-1)(8x-3)=0? Which ordered pairs are solutions to the system? Check all of the boxes that apply. (-1, 10) (-0.375, -5.25) (0.375, 12.75) (1, 0.375) (1, 14) What determines the amount of inertia an object has Choose the correct answer to the question. ?Contar Miguel el dinero? No, no lo contar. No, no lo contar. No, no lo contars. No, no lo contaremos.\ Write the verbs in the right tense formTo many people the word "Hollywood" (1) _____ 2 meanings (to have). Hollywood is an area in Los Angeles. Hollywood (2) _____also the American movie industry (to be).It was just farmland at the beginingof the20th century.Early American movies (3) _____ in other places, such as New York and Chicago (to be made). In 1917 a director was making a film in Chicago.Becouse of cold wather he couldn't finish the film. He took a trip to southern California and (4) _____ it was the perfect place for making films (to realize).Next year his company built a movie studio in Hollywood. Other companies (5)_____ his example (to follow) Which is the central element for all living things?